6
$\begingroup$

Let $A$ be a real $n\times n$ matrix and let $\mu_1, \dots, \mu_n$ the (generalized, complex) eigenvalues of $A$. Assume that $$ 0 < \alpha < \mathrm{Re}(\mu_1) < \dots < \mathrm{Re}(\mu_n).$$

I am interested in a lower bound on the eigenvalues of $A^t A$ in relation to $\alpha$.

If $A$ is symmetric, then the lowest eigenvalue of $A^tA$ is bigger than $\alpha^2$. But if $A$ is non-symmetric, then this is not true.

Question: Is there a positive number $C(\alpha)$ of $\alpha$ such that we have: If for a matrix $A$, the above inequality holds, then the smallest eigenvalue of $A^tA$ is bigger than $C(\alpha)$? And what is the best such $C(\alpha)$?

\Edit: For example, for the matrix $$A := \begin{pmatrix} \alpha & 1 \\ 0 & \alpha \end{pmatrix},$$ the matrix $A^t A$ has the smallest eigenvalue $\alpha^2 + \frac{1}{2}\bigl( 1 - \sqrt{4\alpha^2 + 1}\bigr)$, which is always smaller than $\alpha^2$.

\Edit: Fixed a typing error.

$\endgroup$
7
  • $\begingroup$ The best $C(\alpha)$ is $\alpha^2$. For a proof, compute the determinant. $\endgroup$ Jul 19, 2013 at 16:55
  • 1
    $\begingroup$ This is false, I believe, see my example above. $\endgroup$ Jul 19, 2013 at 21:12
  • $\begingroup$ You might benefit from Ky-Fan's theorem that for any matrix $A$, $\text{Re}\lambda(A) \prec \lambda(\text{Re}A)$, followed by the Fan-Hoffman theorem: $\lambda_j(Re(A)) \le \sigma_j(A)$ (evals and svals are sorted in decreasing order). $\endgroup$
    – Suvrit
    Jul 19, 2013 at 23:13
  • $\begingroup$ Kofi: in your question you are asking for a lowerbound of the biggest eigenvalue. If the question is about the smallest, replace 1 by a big x in your example. $\endgroup$ Jul 20, 2013 at 7:44
  • $\begingroup$ Yes sorry, I fixed that mistake. But what happens if I replace the 1 with a large X in the example? If you can answer the question, I would like to invite you to just post an answer! $\endgroup$ Jul 20, 2013 at 7:52

1 Answer 1

5
$\begingroup$

No, there is no such $C(\alpha)$, even for fixed $n>1$.

Indeed, consider the matrix $A = \begin{pmatrix} \alpha & x \\ 0 & \alpha\end{pmatrix}$.

The product of the eigenvalues of $A^t A$ does not depend on $x$ (it is equal to $det(A)^2=\alpha^4$), whereas their sum goes to $\infty$ (it is $Tr(A^tA)=2\alpha^2+x^2$). Hence one the the eigenvalues goes to $\infty$, and the other to $0$.

$\endgroup$

Your Answer

By clicking “Post Your Answer”, you agree to our terms of service and acknowledge you have read our privacy policy.

Not the answer you're looking for? Browse other questions tagged or ask your own question.